Linear Algebra - Intersection of Affine Spaces












1












$begingroup$


Let V be a vector space, $W_1, W_2$ are sub-spaces of $V$.
$v_1, v_2 in V$ and $(v_1 + W_1) cap(v_2 + W_2) neq emptyset$.



Prove that $(v_1 + W_1) cap(v_2 + W_2)$ is an affine space, i.e. there exists a sub-space $W_3$ of $V$ and $v_3 in V$ so that $(v_1 + W_1) cap(v_2 + W_2) = v_3 + W_3 $.



I have found this previous question but I couldn't figure out the next steps of proving this.



We know that $exists x in (v_1 + W_1) cap(v_2 + W_2) $.



I have no clue how to go on from here. I think I can show that since the intersection is not empty, for all $w_1 in W_1, w_2 in W_2 , w_1 = w_2$.



Would appreciate some points and guidelines about how to approach this.










share|cite|improve this question









$endgroup$








  • 1




    $begingroup$
    It should be more or less obvious (especially if you draw a picture) that the choice of 'offset' (i.e., choice of $v_3$) is arbitrary and can be any point in the intersection.
    $endgroup$
    – SvanN
    Dec 24 '18 at 15:44






  • 1




    $begingroup$
    If $xin v_1+W_1$, then $v_1+W_1=x+W_1$ etc.
    $endgroup$
    – Lord Shark the Unknown
    Dec 24 '18 at 15:53
















1












$begingroup$


Let V be a vector space, $W_1, W_2$ are sub-spaces of $V$.
$v_1, v_2 in V$ and $(v_1 + W_1) cap(v_2 + W_2) neq emptyset$.



Prove that $(v_1 + W_1) cap(v_2 + W_2)$ is an affine space, i.e. there exists a sub-space $W_3$ of $V$ and $v_3 in V$ so that $(v_1 + W_1) cap(v_2 + W_2) = v_3 + W_3 $.



I have found this previous question but I couldn't figure out the next steps of proving this.



We know that $exists x in (v_1 + W_1) cap(v_2 + W_2) $.



I have no clue how to go on from here. I think I can show that since the intersection is not empty, for all $w_1 in W_1, w_2 in W_2 , w_1 = w_2$.



Would appreciate some points and guidelines about how to approach this.










share|cite|improve this question









$endgroup$








  • 1




    $begingroup$
    It should be more or less obvious (especially if you draw a picture) that the choice of 'offset' (i.e., choice of $v_3$) is arbitrary and can be any point in the intersection.
    $endgroup$
    – SvanN
    Dec 24 '18 at 15:44






  • 1




    $begingroup$
    If $xin v_1+W_1$, then $v_1+W_1=x+W_1$ etc.
    $endgroup$
    – Lord Shark the Unknown
    Dec 24 '18 at 15:53














1












1








1





$begingroup$


Let V be a vector space, $W_1, W_2$ are sub-spaces of $V$.
$v_1, v_2 in V$ and $(v_1 + W_1) cap(v_2 + W_2) neq emptyset$.



Prove that $(v_1 + W_1) cap(v_2 + W_2)$ is an affine space, i.e. there exists a sub-space $W_3$ of $V$ and $v_3 in V$ so that $(v_1 + W_1) cap(v_2 + W_2) = v_3 + W_3 $.



I have found this previous question but I couldn't figure out the next steps of proving this.



We know that $exists x in (v_1 + W_1) cap(v_2 + W_2) $.



I have no clue how to go on from here. I think I can show that since the intersection is not empty, for all $w_1 in W_1, w_2 in W_2 , w_1 = w_2$.



Would appreciate some points and guidelines about how to approach this.










share|cite|improve this question









$endgroup$




Let V be a vector space, $W_1, W_2$ are sub-spaces of $V$.
$v_1, v_2 in V$ and $(v_1 + W_1) cap(v_2 + W_2) neq emptyset$.



Prove that $(v_1 + W_1) cap(v_2 + W_2)$ is an affine space, i.e. there exists a sub-space $W_3$ of $V$ and $v_3 in V$ so that $(v_1 + W_1) cap(v_2 + W_2) = v_3 + W_3 $.



I have found this previous question but I couldn't figure out the next steps of proving this.



We know that $exists x in (v_1 + W_1) cap(v_2 + W_2) $.



I have no clue how to go on from here. I think I can show that since the intersection is not empty, for all $w_1 in W_1, w_2 in W_2 , w_1 = w_2$.



Would appreciate some points and guidelines about how to approach this.







linear-algebra vector-spaces






share|cite|improve this question













share|cite|improve this question











share|cite|improve this question




share|cite|improve this question










asked Dec 24 '18 at 15:37









TegernakoTegernako

968




968








  • 1




    $begingroup$
    It should be more or less obvious (especially if you draw a picture) that the choice of 'offset' (i.e., choice of $v_3$) is arbitrary and can be any point in the intersection.
    $endgroup$
    – SvanN
    Dec 24 '18 at 15:44






  • 1




    $begingroup$
    If $xin v_1+W_1$, then $v_1+W_1=x+W_1$ etc.
    $endgroup$
    – Lord Shark the Unknown
    Dec 24 '18 at 15:53














  • 1




    $begingroup$
    It should be more or less obvious (especially if you draw a picture) that the choice of 'offset' (i.e., choice of $v_3$) is arbitrary and can be any point in the intersection.
    $endgroup$
    – SvanN
    Dec 24 '18 at 15:44






  • 1




    $begingroup$
    If $xin v_1+W_1$, then $v_1+W_1=x+W_1$ etc.
    $endgroup$
    – Lord Shark the Unknown
    Dec 24 '18 at 15:53








1




1




$begingroup$
It should be more or less obvious (especially if you draw a picture) that the choice of 'offset' (i.e., choice of $v_3$) is arbitrary and can be any point in the intersection.
$endgroup$
– SvanN
Dec 24 '18 at 15:44




$begingroup$
It should be more or less obvious (especially if you draw a picture) that the choice of 'offset' (i.e., choice of $v_3$) is arbitrary and can be any point in the intersection.
$endgroup$
– SvanN
Dec 24 '18 at 15:44




1




1




$begingroup$
If $xin v_1+W_1$, then $v_1+W_1=x+W_1$ etc.
$endgroup$
– Lord Shark the Unknown
Dec 24 '18 at 15:53




$begingroup$
If $xin v_1+W_1$, then $v_1+W_1=x+W_1$ etc.
$endgroup$
– Lord Shark the Unknown
Dec 24 '18 at 15:53










1 Answer
1






active

oldest

votes


















1












$begingroup$

The idea is to prove that $x+W_1 = v_1+W_1$ and $x+W_2=v_2+W_2$. And then it follows that
$(x+W_1)cap (x+W_2)=x+(W_1cap W_2)$.






share|cite|improve this answer









$endgroup$














    Your Answer








    StackExchange.ready(function() {
    var channelOptions = {
    tags: "".split(" "),
    id: "69"
    };
    initTagRenderer("".split(" "), "".split(" "), channelOptions);

    StackExchange.using("externalEditor", function() {
    // Have to fire editor after snippets, if snippets enabled
    if (StackExchange.settings.snippets.snippetsEnabled) {
    StackExchange.using("snippets", function() {
    createEditor();
    });
    }
    else {
    createEditor();
    }
    });

    function createEditor() {
    StackExchange.prepareEditor({
    heartbeatType: 'answer',
    autoActivateHeartbeat: false,
    convertImagesToLinks: true,
    noModals: true,
    showLowRepImageUploadWarning: true,
    reputationToPostImages: 10,
    bindNavPrevention: true,
    postfix: "",
    imageUploader: {
    brandingHtml: "Powered by u003ca class="icon-imgur-white" href="https://imgur.com/"u003eu003c/au003e",
    contentPolicyHtml: "User contributions licensed under u003ca href="https://creativecommons.org/licenses/by-sa/3.0/"u003ecc by-sa 3.0 with attribution requiredu003c/au003e u003ca href="https://stackoverflow.com/legal/content-policy"u003e(content policy)u003c/au003e",
    allowUrls: true
    },
    noCode: true, onDemand: true,
    discardSelector: ".discard-answer"
    ,immediatelyShowMarkdownHelp:true
    });


    }
    });














    draft saved

    draft discarded


















    StackExchange.ready(
    function () {
    StackExchange.openid.initPostLogin('.new-post-login', 'https%3a%2f%2fmath.stackexchange.com%2fquestions%2f3051366%2flinear-algebra-intersection-of-affine-spaces%23new-answer', 'question_page');
    }
    );

    Post as a guest















    Required, but never shown

























    1 Answer
    1






    active

    oldest

    votes








    1 Answer
    1






    active

    oldest

    votes









    active

    oldest

    votes






    active

    oldest

    votes









    1












    $begingroup$

    The idea is to prove that $x+W_1 = v_1+W_1$ and $x+W_2=v_2+W_2$. And then it follows that
    $(x+W_1)cap (x+W_2)=x+(W_1cap W_2)$.






    share|cite|improve this answer









    $endgroup$


















      1












      $begingroup$

      The idea is to prove that $x+W_1 = v_1+W_1$ and $x+W_2=v_2+W_2$. And then it follows that
      $(x+W_1)cap (x+W_2)=x+(W_1cap W_2)$.






      share|cite|improve this answer









      $endgroup$
















        1












        1








        1





        $begingroup$

        The idea is to prove that $x+W_1 = v_1+W_1$ and $x+W_2=v_2+W_2$. And then it follows that
        $(x+W_1)cap (x+W_2)=x+(W_1cap W_2)$.






        share|cite|improve this answer









        $endgroup$



        The idea is to prove that $x+W_1 = v_1+W_1$ and $x+W_2=v_2+W_2$. And then it follows that
        $(x+W_1)cap (x+W_2)=x+(W_1cap W_2)$.







        share|cite|improve this answer












        share|cite|improve this answer



        share|cite|improve this answer










        answered Dec 24 '18 at 15:53









        MirceaMircea

        1736




        1736






























            draft saved

            draft discarded




















































            Thanks for contributing an answer to Mathematics Stack Exchange!


            • Please be sure to answer the question. Provide details and share your research!

            But avoid



            • Asking for help, clarification, or responding to other answers.

            • Making statements based on opinion; back them up with references or personal experience.


            Use MathJax to format equations. MathJax reference.


            To learn more, see our tips on writing great answers.




            draft saved


            draft discarded














            StackExchange.ready(
            function () {
            StackExchange.openid.initPostLogin('.new-post-login', 'https%3a%2f%2fmath.stackexchange.com%2fquestions%2f3051366%2flinear-algebra-intersection-of-affine-spaces%23new-answer', 'question_page');
            }
            );

            Post as a guest















            Required, but never shown





















































            Required, but never shown














            Required, but never shown












            Required, but never shown







            Required, but never shown

































            Required, but never shown














            Required, but never shown












            Required, but never shown







            Required, but never shown







            Popular posts from this blog

            Plaza Victoria

            In PowerPoint, is there a keyboard shortcut for bulleted / numbered list?

            How to put 3 figures in Latex with 2 figures side by side and 1 below these side by side images but in...